The measures of the angles of a triangle are shown in the figure below. Solve for x.
(6x-11)⁰
23°

PLS HURRY PLS !! :(

The Measures Of The Angles Of A Triangle Are Shown In The Figure Below. Solve For X.(6x-11)23 PLS HURRY

Answers

Answer 1

Answer:

x = 13°

Step-by-step explanation:

-> all angles of triangle add up to 180°

-> right angle = 90°

-> GIVEN: 90° and 23°

90° + 23° + (6x-11) = 180°

6x-11 = 67°

6x = 78°

[tex]\frac{6x}{6} =\frac{78}{6}[/tex]

x = 13°

Answer 2
Sum of the interior angles in a triangle = 180

So, 23 + 90 (right angle) + (6x-11) = 180

23 + 90 + 6x - 11 = 180

Simplify

102 + 6x = 180

Subtract 102 from both sides

6x = 78

Divide by 6

x = 13

Answer: x = 13

Related Questions

sin 30° = 0.5 Using the equality above, copy and complete the following: sin-¹ (0.5) =​

Answers

However, sin⁻¹ is defined to return an angle between -90° and 90°, so it returns the angle that is closest to 30° (which is 30° in this case).

What is trigonometry?

Trigonometry is a branch of mathematics that deals with the relationships between the sides and angles of triangles. It is used to solve problems related to geometry, physics, engineering, and many other fields. Trigonometry is based on the study of the six trigonometric functions: sine (sin), cosine (cos), tangent (tan), cosecant (csc), secant (sec), and cotangent (cot). These functions describe the ratios of the lengths of the sides of a right triangle, and can be used to calculate the unknown side lengths or angles of a triangle.

Here,

If sin 30° = 0.5, then sin⁻¹(0.5) is the angle whose sine is 0.5. In other words, we are looking for the angle whose sine is 0.5. Since sin 30° = 0.5, we know that one possible answer is 30 degrees. However, there are other angles that also have a sine of 0.5. One way to find the other angles is to use the inverse sine function, denoted as sin⁻¹. This function takes a value between -1 and 1 as its input and returns an angle between -90° and 90° as its output. So, if we want to find sin⁻¹(0.5), we are asking: what angle has a sine of 0.5?

The answer is: sin⁻¹(0.5) = 30°.

Note that there are other angles that also have a sine of 0.5, such as 150°, 390°, etc.

To know more about trigonometry,

https://brainly.com/question/26719838

#SPJ1

Complete question:

Using the equality above, copy and complete the following:

The value of sin⁻¹ (0.5)​ when sin 30° = 0.5.

. Calculate the slope of the line that passes through (3, 2) and (-7, 4).

Answers

Answer:

-0.2

Step-by-step explanation:

[tex]\frac{y2-y1}{x2-x1}[/tex]

^This here is how I calculated the slope^

Y2=4

Y1= 2

4-2= 2

X2=-7

x1=3

-7-3=-10

2/-10

or -2/10

how mang triangles are possible given the following side maesurment: 3 feet , 5 feet, 4 feet

Answers

The answer is: 1 triangle is possible  given the following side maesurment: 3 feet , 5 feet, 4 feet.

To determine how many triangles are possible with these side measurements, we can use the triangle inequality theorem, which states that for any triangle, the sum of the lengths of any two sides must be greater than the length of the remaining side.

What is inequality theorem?

In this case, we have three side measurements: 3 feet, 5 feet, and 4 feet. Let's call these sides a, b, and c, respectively. Using the triangle inequality theorem, we can see that:

a + b > c

a + c > b

b + c > a

Substituting in the values of a, b, and c, we get:

3 + 5 > 4

3 + 4 > 5

4 + 5 > 3

All three of these inequalities are true, so it is possible to form a triangle with these side measurements.

To determine how many distinct triangles are possible, we can use the fact that any two triangles are distinct if and only if they have at least one side with a different length. In this case, all three sides have different lengths, so there is only one distinct triangle that can be formed with these side measurements.

Therefore, the answer is: 1 triangle.

To know more about triangle, visit:

https://brainly.com/question/2773823

#SPJ1

Complete question is: 1 triangle is possible given the following side maesurment: 3 feet , 5 feet, 4 feet.

Find the value of X in the isosceles trapezoid

Answers

In an isosceles trapezoid, the diagonals are congruent, so the value of X is 7.

What is isosceles trapezoid?

In an isosceles trapezoid, the two angles formed by each leg and a base are congruent, and the diagonals that connect opposite vertices are congruent as well.

According to question:

An isosceles trapezoid is a four-sided polygon with two parallel sides that are of equal length and two non-parallel sides that are also of equal length. The non-parallel sides are often referred to as the legs of the trapezoid, and the parallel sides are called the bases.

In an isosceles trapezoid, the diagonals are congruent, so we can set KM and JL equal to each other and solve for x:

KM = JL

2x+10 = 5x-11

Subtracting 2x and adding 11 to both sides, we get:

21 = 3x

x = 7

Therefore, the value of x in the isosceles trapezoid is 7.

To know more about trapezoid visit:

https://brainly.com/question/4333941

#SPJ1

Solve for x. Round to the nearest tenth, if necessary.
D
59⁰
7.7
E
Xx
F
Click her

Answers

The value of x in the right triangle when calculated is approximately 13.8 units

Calculating the value of x in the triangle

Given the right-angled triangle

The side length x can be calculated using the following sine ratio

So, we have

sin(39) = x/22

To find x, we can use the fact that sin(39 degrees) = x/22 and solve for x.

First, we can use a calculator to find the value of sin(39 degrees), which is approximately 0.6293.

Then, we can set up the equation:

0.6293 = x/22

To solve for x, we can multiply both sides by 22:

0.6293 * 22 = x

13.8446 = x

Rewrite as

x = 13.8446

Approximate the value of x

x = 13.8

Therefore, x is approximately 13.8 in the triangle

Read about right triangle at

https://brainly.com/question/2437195

#SPJ1

which of the following fractions are equivalent to 15/21 ? select all that apply. a. 5/7 b. 30/42 c. 21/15 d. 25/31 e. 45/84

Answers

The fractions a. 5/7 b. 30/42 e. 45/84 are equivalent to 15/21.

       When it comes to fractions, equivalent means that both fractions show the same part of a whole. The numerator and denominator of equivalent fractions may be multiplied or divided by the same number or the number multiplied by the numerator and denominator should be the same.

 The following are steps to simplify fractions:

 First, find the greatest common factor (GCF) of both numbers.

And then divide both numbers by the GCF. The result is the simplified fraction.

            The greatest common factor of 15 and 21 is 3. By dividing both 15 and 21 by 3, the simplified fraction will be found.

                   15/21 = 5/7

 By dividing both 30 and 42 by 6, the simplified fraction will be found.

                     30/42 = 5/7

 By dividing both 45 and 84 by 3, the simplified fraction will be found.

       45/84 = (5/12)21/15 and 25/31 are not equivalent to 15/21.

     Therefore, the correct options are a. 5/7 b. 30/42 e. 45/84

Learn more about fractions equivalent at: https://brainly.com/question/17220365

#SPJ11

Geometry, and Slope. Worth 15 Points
Please Explain

Answers

The equation of the line perpendicular to the line QR in slope intercept is y= [tex]\frac{1}{2}x+\frac{7}{2}[/tex].

Given,

x+2y=2

y = 1 - [tex]\frac{x}{2}[/tex]

Slope = -1/2 = [tex]m_{1}[/tex]

Let Slope of line perpendicular to line x+2y=2 is [tex]m_{2}[/tex]

⇒  [tex]m_{1}[/tex] *    [tex]m_{2}[/tex] = -1

Then [tex]m_{2}[/tex] = 1/2

Equation of line perpendicular is given by

[tex]y = \frac{1}{2}x + c[/tex]

Since it contains point (5,6)

6 = [tex]\frac{1}{2} (5) + c[/tex]

c= [tex]\frac{7}{2}[/tex]

Then the equation of line is:

[tex]y = \frac{1}{2}x+ \frac{7}{2}[/tex].

Therefore, the equation of the line perpendicular to the line QR in slope intercept is y= [tex]\frac{1}{2}x+\frac{7}{2}[/tex].

Learn more about Equations:

https://brainly.com/question/1401516

#SPJ1

suppose that the value of a stock varies each day from $8.82 to $16.17 with a uniform distribution. find the third quartile, i.e., 75% of all days the stock is below what value?

Answers

The third quartile is $10.6575, which means that 75% of all days the stock is below this value.

The range of the stock price is from $8.82 to $16.17, and the distribution is uniform, which means that the probability of the stock price being any value between the minimum and maximum is the same.

To find the third quartile, we need to find the value x such that 75% of the observations are less than or equal to x, and 25% of the observations are greater than or equal to x.

Since the distribution is uniform, we can find x by finding the value that separates the bottom 25% of the distribution from the top 75% of the distribution.

The bottom 25% of the distribution spans from $8.82 to some value x. Since the distribution is uniform, we can find x by setting the probability of the stock price being less than or equal to x to 0.25, which gives:

(x - 8.82) / (16.17 - 8.82) = 0.25

Solving for x, we get:

x - 8.82 = 0.25 * (16.17 - 8.82)

x - 8.82 = 1.8375

x = 10.6575

Learn more about  descriptive statistics:https://brainly.com/question/6990681

#SPJ11

Please help i really need help due tomorrow

Answers

The area of the composite figure is 80ft².

Define area of composite figure?

Composite shapes may have overlaps between their perimeter and area.

Although we frequently define any shape's area through its perimeter, these two ideas are distinct. The area determines how much room the shape can store, while the perimeter merely draws the object's exterior border. Hence, the space that a shape encloses within its perimeter or boundary is its area.

Calculating the areas of various fundamental shapes is necessary to determine the area of a composite shape.

Breaking the form down is the easiest method:

Separate the composite shape into its constituent parts.

Each fundamental shape's area should be determined separately.

Add these areas together to determine the composite shape's area.

Here, first the area of the triangle:

1/2 × b × h

=1/2 ×(18-7-7) × 4

= 1/2 × 4 × 4

= 8ft².

Now, area of the rectangle:

b × l

= 4× 18

= 74ft².

Area of the whole figure = 8 + 72 = 80ft².

To know more about area, visit:

https://brainly.com/question/16890298

#SPJ1

What is the equation of the line of reflection that reflects shape P into shape Q

Answers

The equation of the line of reflection that reflects shape P into shape Q is y = −2x + 12.

To find the equation of the line of reflection that reflects shape P into shape Q, we need to follow some steps:

Step 1: Draw the mirror line. To reflect a point or shape, we must have a mirror line. The mirror line is the line that passes through the reflection and is perpendicular to the reflecting surface. It serves as a reference for reflecting points or shapes.

Step 2: Find the midpoint of PQ. The midpoint of PQ is the point that lies exactly halfway between P and Q.

Step 3: Find the slope of PQ. The slope of PQ is the rise over run or the difference of the y-coordinates over the difference of the x-coordinates.

The slope formula is given by m = (y2 − y1) / (x2 − x1).

Step 4: Find the perpendicular slope of PQ. The perpendicular slope of PQ is the negative reciprocal of the slope of PQ. It is given by m⊥ = −1/m.

Step 5: Write the equation of the line of reflection. The equation of the line of reflection is given by y − y1 = m⊥(x − x1) or y = m⊥x + b, where m⊥ is the perpendicular slope of PQ and b is the y-intercept of the line. To find b, we substitute the coordinates of the midpoint of PQ into the equation and solve for b. Then we substitute m⊥ and b into the equation to get the final answer.

for such more question on reflection

https://brainly.com/question/26642069

#SPJ11

A 90 digit number 9999. Is divided by 89, what is the remainder?

Answers

The remainder when a 90-digit number 9999 is divided by 89 is 0, as the result of applying the divisibility rule of 89, which involves reversing the digits of the number and subtracting the smaller from the larger.

To find the remainder when a 90-digit number 9999 is divided by 89, we can use the divisibility rule of 89. The rule states that for any integer n, the number obtained by reversing the digits of n and subtracting the smaller from the larger is divisible by 89.

In this case, we reverse the digits of 9999 to get 9999 again, and subtract the smaller from the larger to get 0. Since 0 is divisible by any number, including 89, the remainder when 9999 is divided by 89 is 0.

To know more about divisibility:

https://brainly.com/question/11323048

#SPJ4

how much of a 12% 12 % salt solution must combined with a 26% 26 % salt solution to make 2 2 gallons of a 20% 20 % salt solution?

Answers

To make 2 gallons of a 20% salt solution, combine 0.86 gallons of the 12% salt solution and 1.14 gallons of the 26% salt solution.

Let x be the amount of the 12% salt solution needed in gallons, and y be the amount of the 26% salt solution needed in gallons to make 2 gallons of a 20% salt solution.

Based on the provided data, we can construct the following system of two equations:

X + y = 2 (total volume of the mixture is 2 gallons)

0.12x + 0.26y = 0.2(2) (total salt content of the mixture is 20% of 2 gallons)

Simplifying the second equation, we get:

0.12x + 0.26y = 0.4

Multiplying the first equation by 0.12 and subtracting it from the second equation, we get:

0.14y = 0.16

Y = 1.14

Substituting y = 1.14 into the first equation, we get:

X + 1.14 = 2

X = 0.86

In order to create 2 gallons of a 20% salt solution, 0.86 gallons of the 12% salt solution and 1.14 gallons of the 26% salt solution must be combined.

The complete question is:-

How much of a 12% salt solution must combined with a 26% salt solution to make 2 gallons of a 20% salt solution?

To learn more about solution, refer:-

https://brainly.com/question/30665317

#SPJ4

Evaluate
8


1
+
0. 5

8a−1+0. 5b8, a, minus, 1, plus, 0, point, 5, b when

=
1
4
a=
4
1

a, equals, start fraction, 1, divided by, 4, end fraction and

=
10
b=10b, equals, 10

Answers

The evaluation of the algebraic expression 8a-1+0.5b when a=1/4 and b=10 is 6.

We are given an expression 8a−1+0.5b8, and we are asked to evaluate it when a = 1/4 and b = 10. Substituting these values, we get:

8(1/4) - 1 + 0.5(10) = 2 - 1 + 5

Simplifying the expression on the right-hand side, we get:

8a−1+0.5b8 = 6

Therefore, when a = 1/4 and b = 10, the value of the expression 8a−1+0.5b8 is 6.

This means that if we substitute the given values for a and b, the resulting expression will have a value of 6. It is important to correctly substitute the values before evaluating the expression to obtain the correct answer.

To know more about algebraic expression:

https://brainly.com/question/953809

#SPJ4

____The given question is incomplete, the complete question is given below:

Evaluate 8 a − 1 + 0.5 b 8a−1+0.5b8, a, minus, 1, plus, 0, point, 5, b when a = 1 4 a= 4 1 a, equals, start fraction, 1, divided by, 4, end fraction and b = 10 b=10b, equals, 10.

given the following exponential function, identify whether the change represents growth or decay and determine the percentage rate of increase or decrease y=620(0.941)x

Answers

the function represents exponential decay with a rate of decrease of 5.9% per unit increase in x.

In the exponential function y = [tex]620(0.941)^x:[/tex]

The base of the exponent is 0.941, which is between 0 and 1.

As x increases, the value of [tex](0.941)^x[/tex]gets smaller and smaller, approaching 0 but never reaching it.

Therefore, the function represents exponential decay.

To determine the percentage rate of decrease, we can use the formula:

rate of decrease = (1 - base) x 100%

In this case, the base is 0.941, so the rate of decrease is:

rate of decrease = (1 - 0.941) x 100% = 5.9%

The exponential function is y = 620(0.941)^x.
To determine whether the function represents growth or decay, we need to look at the base of the exponential function, which is 0.941. Since this base is less than 1, the function represents decay.
To determine the percentage rate of decrease, we can use the formula:

r = (1 - b) x 100%
where r is the percentage rate of decrease, and b is the base of the exponential function.
In this case, b = 0.941, so we have:

r = (1 - 0.941) x 100%
= 0.059 x 100%
= 5.9%

Therefore, the exponential function y = 620(0.941)^x represents decay with a rate of 5.9% per unit of x.

A sort of mathematical function called exponential decay can be used to explain a quantity's decline across time or space. The quantity at any given time will change at a pace that is proportionate to the quantity itself, which is characterised by a decreasing rate of change. In other words, the amount of reduction decreases as time or space grows, but it never decreases to zero.

Given the following exponential function, identify whether the change represents growth or decay, and determine the percentage rate of increase or decrease. y=620(0.941)^x y=620(0.941) x

Learn more about exponential decay  here:

https://brainly.com/question/2193799

#SPJ1

five observations taken for two variables follow. xi4611316 yi5050406030 what does the scatter diagram indicate about the relationship between the two variables?

Answers

If we see the scatter plot we can conclude that the possible relation between x and y is linear and with a positive correlation since when the values of x increase the values for y increases as well.

[tex]Cov(x,y) =\frac{\sum_1^n(x_i-X')(y_i-Y')}{n-1}[/tex]

 [tex]\sum_1^5(6-16)(6-10)+(11-16)(9-10)....(27-16)(12-10)=106\\\\and\\Cov(x,y)=\frac{106}{4}=26.5\\\\r=0.693[/tex]

For this part we use excel in order to create the scatterplot and we got the result on the figure attached

If we see the scatter plot we can conclude that the possible relation between x and y is linear and with a positive correlation since when the values of x increase the values of y increase as well

The correlation coefficient is a "statistical measure that calculates the strength of the relationship between the relative movements of two variables". It's denoted by r and its always between -1 and 1.

And in order to calculate the correlation coefficient we can use this

[tex]Cov(x,y) =\frac{\sum_1^n(x_i-X')(y_i-Y')}{n-1}[/tex]

:  

[tex]Cov(x,y) =\frac{\sum_1^n(x_i-X')(y_i-Y')}{n-1}[/tex]

 [tex]\sum_1^5(6-16)(6-10)+(11-16)(9-10)....(27-16)(12-10)=106\\\\and\\Cov(x,y)=\frac{106}{4}=26.5\\\\r=0.693[/tex]

learn more about  scatter plot

https://brainly.com/question/29231735

#SPJ4

A line has a slope of 0 and passes through the point (4,4). Write its equation in slope-intercept form.

Answers

Answer:The equation of a line that passes through a point is an algebraic equation. It can also be referred to as the Slope-Intercept Equation.The equation of the line that passes through the point (4, 4) and has a slope of 0 is written as: y = 4 The equation of the line through a point (x1, y1) can be represented by the algebraic equation:y = mx + cwhere:m = slopec = y - interceptFrom the question,(x1, y1) = (4, 4)m = slope = 0Substituting these values into the algebraic equation,4 = (0 x 4) + c Hence, y = 4The equation of the line that passes through the point (4, 4) and has a slope of zero is y = 4

Answer:

y=4

Step-by-step explanation:

If a line has a slope of 0, then it is a horizontal line, and all points on the line have the same y-coordinate. We are given that the line passes through the point (4, 4). Therefore, the equation of the line in slope-intercept form is:

y = b

where b is the y-coordinate of the point through which the line passes. In this case, b = 4, so the equation of the line is:

y = 4

Therefore, the equation of the line in slope-intercept form is y = 4.

(Please could you kindly mark my answer as brainliest)

solve for x and find it​

Answers

[tex]\frac{65x-14x+49-4}{14} = 36[/tex]The value of x is 9.

What is inquality?

Inequality refers to a situation where there is a significant difference in the distribution of resources, opportunities, and power among individuals or groups in a society. It is a complex social and economic phenomenon that can manifest in various forms such as income inequality, wealth inequality, gender inequality, racial inequality, educational inequality, and healthcare inequality, among others.

Given by the question.

[tex]\frac{9x+7}{2} - \frac{7x-x+2}{7} = 36[/tex]

[tex]\frac{63x+44-14x+2x+4}{14} = 36[/tex]

[tex]\frac{65x-14x+49-4}{14} = 36[/tex]

[tex]51x+45=504\\51x= 459\\x=9[/tex]

To learn more about inquality:

https://brainly.com/question/29072717

#SPJ1

PLS HELP! WILL MAKE U BRAINLIST

Answers

Answer:

(5,2)

Step-by-step explanation:

Let's solve your system by substitution.

[tex]x+y=7{\text{ ; }}x=y+3[/tex]

Step 2: let Solve [tex]$x+y=7$[/tex] for[tex]$x$[/tex]

[tex]x+y=7[/tex]

[tex]x+y +(-x)=7+(-x)[/tex] (Add (-x) on both sides)

[tex]y=-x+7[/tex]

0+(x)=7-x-y+(x)   (Add (x) on both sides)

x = -y + 7

x/1 = -y+7/1  (divide through by 1)

x = -y + 7

Substitute -y+7 for x in x = y + 3, then solve for u

(-y + 7) = y + 3

-y + 7 = y + 3 (simplify)

-y+7+(-7) = y + 3 + (-7)    (Add (-7) on both sides)

-y=y-4

-y = y-4 (simplify)

-y+(-y)=y-4+(-y)  (Add (-y) on both sides)

-2y-=-4

-2y/-2 = -4/-2  (Divide through by -2)

y = 2

Substitute in 2 for y in x = -y + 7

x =  -y+7

x = -2+7

x = 5

Answer:

x = 5 and y = 2

The table shows four transactions (in dollars) for a bank account. Positive numbers represents deposits, and negative numbers represent withdrawals. The balance before the transactions is $75.50. What could the transaction for 11/7 be if the final balance in the bank account is $86.92? Date Amount 11/4 50.68 11/4 -22.16 11/7 11/11 56.65 End Balance 86.92

Answers

Answer:

11/7: +9.42

Step-by-step explanation:

what are the advantages of a best-guess (trial and error) experiment versus a factorial or design experiment

Answers

One advantage of best-guess experiments is that they are often faster and more cost-effective than factorial or design experiments.

Best-guess (trial and error) experiments involve making a hypothesis and testing it through a series of trials until a satisfactory result is achieved. On the other hand, factorial or design experiments involve manipulating multiple variables simultaneously to determine their individual and interactive effects on a response variable.

Both approaches have their advantages and disadvantages depending on the specific research question and goals. They may also be useful in situations where there is limited knowledge about the variables of interest or when the system is too complex to be modeled accurately.

However, best-guess experiments may suffer from issues such as biased or subjective interpretation of results, a lack of control over extraneous variables, and a potential for false positives or negatives.

In contrast, factorial or design experiments provide a more systematic approach to testing hypotheses and offer greater control over variables, leading to more reliable and generalizable results. They may, however, be more time-consuming and expensive to conduct.

Ultimately, the choice between best-guess and factorial or design experiments depends on the research question, available resources, and desired level of precision and control.

To learn more about experiment click on,

https://brainly.com/question/30455826

#SPJ4

1. if you repeated a hypothesis test 1000 times (i.e. 1000 different samples from the same population), how many times would you expect to commit a type i error, assuming the null hypothesis were true, if:

Answers

If we repeated a hypothesis test 1000 times, the number of times we would expect to commit a Type I error, assuming the null hypothesis were true, would depend on the significance level (α) of the test.

A Type I error occurs when we reject the null hypothesis when it is actually true. The significance level of a test (α) is the probability of making a Type I error when the null hypothesis is true. In other words, if we set a significance level of α = 0.05, we are saying that we are willing to tolerate a 5% chance of making a Type I error.

Assuming a significance level of α = 0.05, if we repeated the test 1000 times, we would expect to make a Type I error in approximately 50 tests (0.05 x 1000 = 50). This means that in 50 out of the 1000 tests, we would reject the null hypothesis even though it is actually true.

However, it is important to note that the actual number of Type I errors we make in practice may differ from our expectation, as it depends on the specific characteristics of the population being tested and the sample sizes used in each test.

For more details about hypothesis click here:

https://brainly.com/question/29519577#

#SPJ11

A town has a population of 12,000 and grows at 3. 5% every year. What will be the population after 7 years, to the nearest whole number?

Answers

If the population growth rate is 3.5 percent every year then the population of the town after 7 years would be 14940.

Given that population grows 3.5 percent every year.

So, the increase in population after one year

= 3.5% of 12000

= (3.5/100) × 12000

= 420

Thus the increase in population after 7 year would be,

= population increase in one year × 7

= 420×7 = 2940

Hence population of the town after 7 years = (present population + increase in population)

= 12000 + 2940

= 14940

So the population of the town after 7 years with 3.5 % growth every year would be 14490.

Visit here to learn more about the percent: https://brainly.com/question/28670903

#SPJ4

Distributive property applied to 7(2x+5) - 4(2x+5)

Answers

To implement the distributing principle to the phrase 7(2x+5) - 4(2x+5), we must first divide the 7 and 4 within the parenthesis to their respective terms:

7(2x+5) - 4(2x+5) = (72x + 75) - (42x + 45)

Within the parenthesis, each term is simplified:

= (14x + 35) - (8x + 20)

We may now reduce the phrase by grouping similar terms:

= 14x - 8x + 35 - 20

= 6x + 15

As a consequence, using the distributive property on 7(2x+5) - 4(2x+5) yields 6x + 15.

By expansion or multiplying, the distribution principle is frequently employed to compress statements and solve problems. It enables us to translate complicated statements into shorter language and conversely. The distributive principle is commonly utilized in mathematics, mathematics, as well as other mathematical subjects.

To know more about multiplying click here

brainly.com/question/25114566

#SPJ9

A line passes through the point (-8, 7) and has a slope of -5/4
Write an equation in slope-intercept form for this line.

Answers

The slope-intercept form of the equation of a line is y = mx + b, where m is the slope and b is the y-intercept.

We are given that the line passes through the point (-8, 7) and has a slope of -5/4. So we can substitute these values into the slope-intercept form and solve for b:

y = mx + b

7 = (-5/4)(-8) + b

7 = 10 + b

b = -3

Therefore, the equation of the line in slope-intercept form is:

y = (-5/4)x - 3

5. Use the digits 3, 4, 5, 6, 7 and 8 to complete the statement.​

Answers

Answer:

Step-by-step explanation:

i don't see a statement but use 7 i guess


A store sells boxes of juice is equal size packs. Garth bought 18 boxes, Rico bought 36 boxes and Mia bought 45 boxes. What is the greatest number of boxes in each pack? How many packs did each person buy if each box contained the greatest number of boxes?

Answers

Answer:29160

Step-by-step explanation:

Compare using <, >, or =.
3 yards
10 feet

Answers

Answer: 10 feet > 3 yards

Step-by-step explanation:

if 1 yard = 3 feet

then 3 yards = 9 feet

so 10 feet > 3yards

at the end of 2024, marin co. has accounts receivable of $673,200 and an allowance for doubtful accounts of $24,010. on january 24. 2025, it is learned that the company's receivable from madonna inc. is not collectible and therefore management authorizes a write- off of $4.147.

Answers

The write-off of the receivable from Madonna Inc. is a necessary adjustment to ensure the accuracy of Marin Co.'s financial statements. Without it, the company's accounts receivable would be overstated and their financial statements would not provide an accurate portrayal of the company's financial position.

At the end of 2024, Marin Co. had accounts receivable of $673,200 and an allowance for doubtful accounts of $24,010. On January 24th, 2025, it was determined that the company's receivable from Madonna Inc. was not collectible and management authorized a write-off of $4,147. This action reduces the accounts receivable balance by $4,147, and reduces the allowance for doubtful accounts by the same amount. The net effect on the balance sheet is a reduction of $4,147 in both accounts receivable and allowance for doubtful accounts.

The impact of the write-off on the company's financial statements is a decrease in net income for the period. This is because a write-off is recognized as an expense, which reduces the amount of net income reported in the period. The amount of the write-off is recorded as an expense on the income statement. In this case, the amount of the write-off is $4,147.

The journal entry to record the write-off would be: Accounts Receivable 4,147; Allowance for Doubtful Accounts 4,147. This entry reduces the accounts receivable and allowance for doubtful accounts by $4,147. The write-off of $4,147 is recorded as an expense on the income statement, and this reduces the net income reported for the period.

for such more questions on financial statements

https://brainly.com/question/26240841

#SPJ11

Help me match themmm

Answers

The volume for each cylinder, given the dimensions, is as follows:

1. r = 4 units, h = 6 units, V = 96π units².

2. r = 8 units , h = 3 units , V = 192π units².

3. r = 1 unit(half of the diameter), h = 5 units, V = 5 units².

4. r = 5 units, h = 7 units, V = 175π units².

How to obtain the volume of a cylinder?

The volume of a cylinder of radius r and height h is given by the equation presented as follows, in which the square of the radius is multiplied by π and the height, hence:

V = πr²h.

Then the equation is applied to each option in this problem with the given dimensions to obtain the volumes.

More can be learned about volume at brainly.com/question/12004994

#SPJ1

Which of the following shows an example of two irrational numbers being multiplied to get a rational number?
Responses

3×9

0×5√

2√ ×8√

2√×3√

Answers

Step-by-step explanation:

Which of the following shows an example of two irrational numbers being multiplied to get a rational number?

Responses

option c

Other Questions
The volume of a gas is 200 mL at 350.0 kPa pressure. What will the volume be when the pressure is reduced to 125.0 kPa, assuming the temperature remains constant. the empirical formula of a chemical substance is ch2. the molar mass of a molecule of the substance is 56.108 g/mol. what is the molecular formula of the chemical substance? (4 points) c3h4 c4h8 c2h4 c6h6 HELP, PLEASEEEEE :(The dimensions of a triangular prism are shown below5cm12cm13cm8cmWhat is the total surface area of the prism in square centimeters? What does proper netiquette mean? A. Talking to others the same way thatthey're talking to youB. Treating others as you want to betreated C. Establishing dominance early on D. Building the structures needed forothers to have Internet access DLFDM = 8MKFM = 2xEEM = 6 Find the x- and y-intercepts of the function f(x) = log(2x + 1) 1.The x-intercept of the function f(x) = log(2x + 1) 1 is . Its y-intercept is 10. What best describes the way Henri viewed hedgehogs after he saw them eating the ants? which is true? group of answer choices a reference declaration and object creation can be combined in a single statement three references can not refer to the same object the new operator is used to declare a reference a reference variable contains data rather than a memory address 12000 x 5 = 9500 + 9500 + 10000 + x + y 49. which statement best describes the term quality assessment (qa)? qa is the overall process of guaranteeing quality patient care. qa is the process of assuring acceptable standards are met for all procedures performed. qa is the review of all quality system essentials (qses). qa is the process of documenting quality control activities. the following data pertains to activity and costs for two months: june july activity level in units 10,000 11,000 direct materials $ 17,000 $ ? fixed factory rent 21,000 ? other production costs 20,000 ? total cost $ 58,000 $ 61,300 assuming that these activity levels are within the relevant range, the other production costs for july were which is a true statement about polygenes? polygenes are located on the same chromosome. polygenes are used to predict the genotype of progeny. polygenes interact and allow for continuous variation. polygenes allow geneticists to maintain any genotype. Does the phrase Survival of the fittest refer to an individual (single organism) or a species (group of same organisms)? Why? How did the prisoners reinforce one another's role as "prisoner"? How did the guards reinforce their role Write an essay about the Uncle Marcos text by Isabel Allende 4. Which of the following is the square of a binomial? A. r - 2rs +s B. c + d C. 16x - 25yD. d - de + e based on what you read in the management in action case, assume that la croix tries to reduce its prices below competitors like pepsico and coca-cola, but targets a narrow market. what type of strategy would this be? Which of these is not an input device?a. Scannerb. Screenc. Keyboardd. Microphone true or false: the president submits his budget to congress in april, where it will undergo varying degrees of change. John drives to work each morning, and the trip takes an average of = 38 minutes. The distribution of driving times is approximately normal with a standard deviation of = 5 minutes. For a randomly selected morning, what is the probability that Johns drive to work will take between 36 and 40 minutes?